Laplace transforms to solve initial value DE / partial fractions

Click For Summary
The discussion focuses on solving the initial value problem using Laplace transforms for the equation f''(y) + 4f'(y) + 8y = u(t-1) with given initial conditions. The user has derived F(Y) but is confused about the next steps, particularly in applying partial fractions to simplify the expression. A suggestion is made to break down F(Y) into its components and tackle each term separately. The importance of correctly factoring the denominator is emphasized, which can aid in applying partial fractions effectively. The conversation highlights common challenges faced when using Laplace transforms for differential equations.
CrazyCamo
Messages
1
Reaction score
0
Hey guys, i have read many posts on physics forums but this would be my first post. I am quite stuck so any help would be much appreciated.

Homework Statement



Use Laplace transforms to solve the initial value problem:

f''(y) + 4f'(y) +8y = u(t-1) where y(0) = 1 and y'(0) = -1

Solve this problem using laplace transforms, showing all steps in your reasoning. State the solution y(t) for each of 0<t<1 and t>1, then sketch it over the range 0<= t <= 10, noting its main features.

Homework Equations





The Attempt at a Solution



I have gotten up to F(Y) = (e^-s + s^2 + 3s)/(s(s^2 + 4s+8))

However, from here i am not sure what to do. I tried taking the partial fractions of:

1/(s(s^2 + 4s+8))

but am getting very confused. Again any help would be much appreciated. Cheers
 
Physics news on Phys.org
Why don't you break F(Y) down into the sum of its various components?

E.g., D = (s(s^2+4s+8))

F(Y) = (e^-s)/D + s^2/D + 3s/D

You can tackle each term individually.

PS: finding the PFE of 1/D doesn't help.
 
The denominator is s(s^2+4s+ 8)= s(s^2+ 4s+ 4+ 4)= s((s+ 2)^2+ 4) so you can use "partial fractions to write that as \frac{A}s+ \frac{Bs+ C}{(s+2)^2+ 4}.
 
Question: A clock's minute hand has length 4 and its hour hand has length 3. What is the distance between the tips at the moment when it is increasing most rapidly?(Putnam Exam Question) Answer: Making assumption that both the hands moves at constant angular velocities, the answer is ## \sqrt{7} .## But don't you think this assumption is somewhat doubtful and wrong?

Similar threads

Replies
9
Views
2K
  • · Replies 6 ·
Replies
6
Views
1K
  • · Replies 1 ·
Replies
1
Views
1K
  • · Replies 7 ·
Replies
7
Views
2K
  • · Replies 4 ·
Replies
4
Views
3K
  • · Replies 8 ·
Replies
8
Views
2K
  • · Replies 8 ·
Replies
8
Views
1K
  • · Replies 2 ·
Replies
2
Views
2K
  • · Replies 10 ·
Replies
10
Views
2K
  • · Replies 4 ·
Replies
4
Views
3K